2019 AMC 10A Problems/Problem 7

Revision as of 00:27, 27 February 2019 by Sevenoptimus (talk | contribs) (Cleaned up the solutions for clarity, removed a redundant one, and reordered the solutions)
The following problem is from both the 2019 AMC 10A #7 and 2019 AMC 12A #5, so both problems redirect to this page.

Problem

Two lines with slopes $\dfrac{1}{2}$ and $2$ intersect at $(2,2)$. What is the area of the triangle enclosed by these two lines and the line $x+y=10  ?$

$\textbf{(A) } 4 \qquad\textbf{(B) } 4\sqrt{2} \qquad\textbf{(C) } 6 \qquad\textbf{(D) } 8 \qquad\textbf{(E) } 6\sqrt{2}$

Solution 1

Let's first work out the slope-intercept form of all three lines: $(x,y)=(2,2)$ and $y=\frac{x}{2} + b$ implies $2=\frac{2}{2} +b=1+b$ so $b=1$, while $y=2x + c$ implies $2= 2 \cdot 2+c=4+c$ so $c=-2$. Also, $x+y=10$ implies $y=-x+10$. Thus the lines are $y=\frac{x}{2} +1, y=2x-2,$ and $y=-x+10$. Now we find the intersection points between each of the lines with $y=-x+10$, which are $(6,4)$ and $(4,6)$. Using the distance formula and then the Pythagorean Theorem, we see that we have an isosceles triangle with base $2\sqrt{2}$ and height $3\sqrt{2}$, whose area is \boxed{\textbf{(C) }6}$.

==Solution 2== Like in Solution 1, we determine the coordinates of the three vertices of the triangle. Now, using the [[Shoelace Theorem]], we can directly find that the area is$ (Error compiling LaTeX. Unknown error_msg)\boxed{\textbf{(C) }6}$.

==Solution 3== Like in the other solutions, solve the systems of equations to see that the triangle's two other vertices are at$ (Error compiling LaTeX. Unknown error_msg)(4, 6)$and$(6, 4)$. Then apply Heron's Formula: the semi-perimeter will be$s = \sqrt{2} + \sqrt{20}$, so the area reduces nicely to a difference of squares, making it$\implies \boxed{\textbf{(C) }6}$.

==Solution 4== Like in the other solutions, we find, either using algebra or simply by drawing the lines on squared paper, that the three points of intersection are$ (Error compiling LaTeX. Unknown error_msg)(2, 2)$,$(4, 6)$and$(6, 4)$. We can now draw the bounding square with vertices$(2, 2)$,$(2, 6)$,$(6, 6)$and$(6, 2)$, and deduce that the triangle's area is$16-4-2-4=\boxed{\textbf{(C) }6}$.

See Also

2019 AMC 10A (ProblemsAnswer KeyResources)
Preceded by
Problem 6
Followed by
Problem 8
1 2 3 4 5 6 7 8 9 10 11 12 13 14 15 16 17 18 19 20 21 22 23 24 25
All AMC 10 Problems and Solutions
2019 AMC 12A (ProblemsAnswer KeyResources)
Preceded by
Problem 4
Followed by
Problem 6
1 2 3 4 5 6 7 8 9 10 11 12 13 14 15 16 17 18 19 20 21 22 23 24 25
All AMC 12 Problems and Solutions

The problems on this page are copyrighted by the Mathematical Association of America's American Mathematics Competitions. AMC logo.png